Find a value of the constant k such that the limit exists

Click For Summary
To find a value of the constant k such that the limit exists for the expression lim (x^2 - kx + 4) / (x - 1) as x approaches 1, k must equal 5. This value allows the polynomial to factor as (x-1)(x-4), which cancels with the denominator, resulting in a limit of -3. A more systematic approach involves recognizing that one root must be 1, leading to the conclusion that k is derived from the relationships between the coefficients of the factors. The discussion also touches on evaluating limits involving exponential functions, emphasizing the importance of understanding behavior as x approaches negative infinity. Overall, the focus is on finding k through factoring and systematic reasoning rather than guessing.
Jacobpm64
Messages
235
Reaction score
0
Find a value of the constant k such that the limit exists.

lim (x2 - kx + 4) / (x - 1)
x->1

We could do...
just try number until it factors nicely..
k would equal 5.. to give us
x2-5x+4 = (x-1)(x-4)
the (x-1) would cancel .. leaving just x-4.. and the limit would be 1-4 = -3...

Is there an easier way of doing this than just guessing to try to figure out which value of k would make the polynomial factor nicely so that it would cancel with the factor in the denominator? Because, some of the other problems in this section get a little too tough to just be able to spit out the answer...
 
Physics news on Phys.org
I'm not sure you could say this is easier. But definitely more systematic.

For the problem you just solved, you can break the function as such:

x^2-kx+4=(x+a)(x+b)=x^2+(a+b)x+ab

To cancel the bottom, you know one of the roots must be 1 and therefore must have a (x-1) factor on top. Hence let a=-1. Also from the first equation, you know that ab = 4. Therefore b=-4. From the equation, you know that -k=(a+b)=-5, so k=5.
 
ok, I understand how to do those now.. But, how would I set up something like this?

lim (e2x - 5) / (ekx + 3)
x-> -infinity
 
Well, try to think about that one logically. What happens to e^2x and e^kx (assuming k is not negative) as x-> -infinity.
 
Question: A clock's minute hand has length 4 and its hour hand has length 3. What is the distance between the tips at the moment when it is increasing most rapidly?(Putnam Exam Question) Answer: Making assumption that both the hands moves at constant angular velocities, the answer is ## \sqrt{7} .## But don't you think this assumption is somewhat doubtful and wrong?

Similar threads

  • · Replies 5 ·
Replies
5
Views
1K
  • · Replies 5 ·
Replies
5
Views
2K
Replies
9
Views
2K
  • · Replies 11 ·
Replies
11
Views
2K
  • · Replies 14 ·
Replies
14
Views
2K
  • · Replies 3 ·
Replies
3
Views
911
  • · Replies 15 ·
Replies
15
Views
2K
Replies
2
Views
1K
Replies
24
Views
3K
  • · Replies 5 ·
Replies
5
Views
2K